Tải bản đầy đủ (.pdf) (69 trang)

ứng dụng nguyên lý dirichlet vào chứng minh bất đẳng thức

Bạn đang xem bản rút gọn của tài liệu. Xem và tải ngay bản đầy đủ của tài liệu tại đây (343.45 KB, 69 trang )

ĐẠI HỌC THÁI NGUYÊN
TRƯỜNG ĐẠI HỌC KHOA HỌC
- - - - - - - - - - - - - - - - - -
Bùi Thị Lan Hương
ỨNG DỤNG NGUYÊN LÝ DIRICHLET VÀO
CHỨNG MINH BẤT ĐẲNG THỨC
LUẬN VĂN THẠC SỸ TOÁN HỌC
Chuyên ngành: PHƯƠNG PHÁP TOÁN SƠ CẤP
Mã số: 60.46.01.13
Người hướng dẫn khoa học
PGS. TS. TRỊNH THANH HẢI
THÁI NGUYÊN - NĂM 2014
1
Mục lục
Mở đầu 2
1 Sơ lược về chứng minh bất đẳng thức 4
1.1 Bất đẳng thức . . . . . . . . . . . . . . . . . . . . . . . . . 4
1.2 Một vài phương pháp chứng minh bất đẳng thức . . . . . . 5
1.2.1 Phương pháp biển đổi tương đương . . . . . . . . . 5
1.2.2 Phương pháp phản chứng . . . . . . . . . . . . . . . 9
1.2.3 Phương pháp quy nạp toán học . . . . . . . . . . . . 11
1.2.4 Sử dụng tam thức bậc hai . . . . . . . . . . . . . . . 12
1.2.5 Sử dụng bất đẳng thức AM-GM . . . . . . . . . . . 16
1.2.6 Sử dụng bất đẳng thức Bunhiacovski . . . . . . . . . 19
1.2.7 Sử dụng bất đẳng thức Karamata . . . . . . . . . . 22
1.2.8 Vận dụng tính chất của hàm số đơn điệu vào chứng
minh bất đẳng thức . . . . . . . . . . . . . . . . . . 24
1.2.9 Vận dụng tính chất hình học vào chứng minh bất
đẳng thức . . . . . . . . . . . . . . . . . . . . . . . 27
2 Ứng dụng nguyên lý Dirichlet vào chứng minh bất đẳng
thức 32


2.1 Tổng quan về nguyên lý Dirichlet . . . . . . . . . . . . . . . 32
2.1.1 Nguyên lý Dirichlet . . . . . . . . . . . . . . . . . . 32
2.1.2 Một số dạng phát biểu nguyên lý Dirichlet . . . . . . 32
2.2 Ứng dụng nguyên lý Dirichlet vào chứng minh bất đẳng thức 35
2.2.1 Ý tưởng . . . . . . . . . . . . . . . . . . . . . . . . 35
2.2.2 Một số ví dụ minh họa . . . . . . . . . . . . . . . . 36
2.2.3 Bài tập tương tự . . . . . . . . . . . . . . . . . . . . 64
Kết luận 67
Tài liệu tham khảo 68
2
Mở đầu
Bất đẳng thức là chuyên đề quen thuộc và quan trọng đối với Toán học
và người làm Toán, học Toán. Các bài toán về bất đẳng thức có mặt trong
hầu hết đề thi học sinh giỏi, Đại học, Olympic Trong phân phối chương
trình chuyên sâu Toán 10 Trung học phổ thông do Bộ giáo dục ấn hành,
ngoài nội dung bắt buộc, chuyên đề bất đẳng thức chiếm khoảng 12 tiết
trong số 55 tiết chuyên đề. Tuy nhiên đây là chuyên đề khó vì đòi hỏi người
làm Toán phải có vốn kiến thức vững vàng, đồng thời linh hoạt, sáng tạo
vận dụng kiến thức khi giải Toán. Cũng chính vì lí do đó mà các bài toán
về bất đẳng thức vô cùng phong phú, đa dạng. Nó khơi gợi óc sáng tạo, tư
duy, góp phần hình thành, củng cố và phát triển năng lực phân tích, giải
quyết vấn đề của người học Toán.
Nguyên lý Dirichlet được phát biểu đầu tiên bởi nhà toán học Đức
Johann Peter Gustav Lejeune Dirichlet (1805-1859) như sau: “nếu nhốt
n + 1 con thỏ vào n cái chuồng thì bao giờ cũng có một chuồng chứa ít
nhất 2 con thỏ”. Đây là phương pháp thông dụng và hiệu quả để giải nhiều
dạng toán.
Hiện nay có một số đề tài đã tìm hiểu và ứng dụng nguyên lí Dirichlet
vào giải một vài dạng toán nhưng chưa có ai tập trung khai thác rõ việc
vận dụng nguyên lý Dirichlet vào chứng minh bất đẳng thức. Mặt khác

vấn đề này đã được nêu trong tạp chí như tạp chí Toán học và Tuổi trẻ,
nhưng mới chỉ dừng ở ý tưởng, vài ví dụ đơn giản và đưa ra các bài tập.
Vì những lí do trên nên tôi chọn đề tài “Ứng dụng nguyên lý
Dirichlet vào chứng minh bất đẳng thức” làm đề tài luận văn Thạc
sĩ.
Luận văn gồm hai nhiệm vụ nghiên cứu chính
• Tổng quan về bất đẳng thức ở phổ thông.
• Vận dụng nguyên lý Dirichlet vào chứng minh một vài bất đẳng thức.
3
Ngoài phần mở đầu và phần kết luận, luận văn gồm hai chương
Chương 1: Sơ lược về chứng minh bất đẳng thức
Nội dung chương 1 giới thiệu vài phương pháp chứng minh bất đẳng
thức kèm theo ví dụ minh họa cho từng phương pháp ở trường phổ thông.
Chương 2: Ứng dụng nguyên lý Dirichlet vào chứng minh bất
đẳng thức
Nội dung chương 2 nêu tổng quan về nguyên lý Dirichlet, ứng dụng
nguyên lý Dirichlet vào chứng minh bất đẳng thức và một số ví dụ minh
họa chứng minh bất đẳng thức bằng cách sử dụng nguyên lý Dirichlet.
Cuối chương 2 là một số ví dụ để độc giả tham khảo và tự chứng minh.
Trong quá trình làm luận văn, chúng tôi đã tham khảo, sử dụng các tài
liệu bồi dưỡng học sinh giỏi, giáo trình của GS.TSKH. Nguyễn Văn Mậu,
Tạp chí Toán học và tuổi trẻ từ năm 1964, đề thi Đại học từ năm 1970
đến nay, một số đề thi Olympic Toán
Tôi xin gửi lời cảm ơn tới Ban Giám hiệu, khoa Toán - Tin, phòng Đào
tạo trường Đại học Khoa học - Đại học Thái Nguyên, các thầy cô giáo
đã trang bị kiến thức và tạo điều kiện tốt nhất cho tôi trong quá trình
học tập và nghiên cứu. Xin bày tỏ lòng cảm ơn chân thành và sâu sắc tới
PGS.TS. Trịnh Thanh Hải, người đã tận tình chỉ bảo, hướng dẫn tôi hoàn
thành luận văn.
Mặc dù luận văn đã hoàn thành nhưng không tránh khỏi những thiếu

sót. Tôi rất mong nhận được những lời đóng góp ý kiến của các thầy cô
giáo và các bạn về những mặt tích cực và hạn chế để luận văn được hoàn
thiện hơn.
Thái Nguyên, tháng 04 năm 2014
Học viên
Bùi Thị Lan Hương
4
Chương 1
Sơ lược về chứng minh bất đẳng
thức
1.1 Bất đẳng thức
• Khái niệm bất đẳng thức
Định nghĩa 1.1. Cho hai số thực a và b. a được gọi là lớn hơn b, kí hiệu
a > b nếu hiệu a − b là một số dương; a được gọi là lớn hơn hoặc bằng b,
kí hiệu a ≥ b nếu hiệu a − b là một số không âm; a được gọi là nhỏ hơn
b, kí hiệu a < b nếu hiệu a − b là một số âm; a được gọi là nhỏ hơn hoặc
bằng b, kí hiệu a ≤ b nếu hiệu a − b là một số không dương.
• Một vài tính chất của bất đẳng thức
Với các số thực a, b, c và số tự nhiên n luôn có tính chất
a > b ⇐⇒ a −b > 0.
a > b ⇐⇒ a + c > b + c.
a > b ⇐⇒ a
2n+1
> b
2n+1
.
| a |>| b | ⇐⇒ a
2n
> b
2n

.
a ≥ b ⇐⇒ a = b hoặc a > b.
Với a > b, c > 0 ⇐⇒ ac > bc.
Với a > b, c < 0 ⇐⇒ ac < bc.
a > b, b > c ⇐⇒ a > c.
| a |≤ α ⇐⇒ α ≥ 0 và −α ≤ a ≤ α.
5
1.2 Một vài phương pháp chứng minh bất đẳng
thức
1.2.1 Phương pháp biển đổi tương đương
• Ý tưởng
- Để chứng minh bất đẳng thức bằng phương pháp biến đổi tương
đương ta sử dụng một số biến đổi sơ cấp để đưa bất đẳng thức cần
chứng minh về bất đẳng thức mới, trong đó bất đẳng thức mới hiển
nhiên đúng hoặc ta có thể chứng minh được.
A > B ⇔ C > D
(trong đó C > D là bất đẳng thức hiển nhiên đúng hoặc ta có thể
chứng minh được.)
- Có 2 phương pháp biến đổi tương đương là biến đổi tương đương trực
tiếp hoặc đặt ẩn phụ rồi biến đổi tương đương.
• Ví dụ minh họa
Ví dụ 1.1 (JMO 2004 - [5]). Cho a, b, c > 0, a + b + c = 1. Chứng minh
rằng
1 + a
1 − a
+
1 + b
1 − b
+
1 + c

1 − c
≤ 2

b
a
+
c
b
+
a
c

. (1.1)
Nhận xét 1.1. Các số hạng ở vế trái đều có dạng
1 + x
1 − x
nên ta viết
1 + x
1 − x
= 1 +
2x
1 − x
.
Giải
Ta biến đổi bất đẳng thức (1.1)
(1.1) ⇔ 3 +
2a
1 − a
+
2b

1 − b
+
2c
1 − c
≤ 2

b
a
+
c
b
+
a
c

⇔ 2a

1
c

1
1 − a

+ 2b

1
a

1
1 − b


+ 2c

1
b

1
1 − c

≥ 3
⇔ a

1
c

1
b + c

+ b

1
a

1
c + a

+ c

1
b


1
a + b


3
2

ab
c(b + c)
+
bc
a(c + a)
+
ca
b(a + b)

3
2
.
6
Ta có


ab
c
+

bc
a

+

ac
b

2
=


ab
c(b + c)
·

b + c +

bc
a(c + a)
·

c + a +

ac
b(a + b)
·

a + b

2
.
Theo bất đẳng thức Bunhiacovski ta có



ab
c(b + c)
·

b + c +

bc
a(c + a)
·

c + a +

ac
b(a + b)
·

a + b

2


ab
c(b + c)
+
bc
a(c + a)
+
ac

b(a + b)

[2(a + b + c)]
nên


ab
c
+

bc
a
+

ac
b

2


ab
c(b + c)
+
bc
a(c + a)
+
ac
b(a + b)

[2 (a + b + c)] .

Theo bất đẳng thức AM-GM ta có
(a + b + c)
2
≥ 3(ab + bc + ca)
nên


ab
c
+

bc
a
+

ac
b

2
≥ 3


ab
c
·
bc
a
+

bc

a
·
ac
b
+

ac
b
·
ab
c

= 3(b + c + a).
Suy ra
3(a + b + c) ≤

ab
c(b + c)
+
bc
a(c + a)
+
ac
b(a + b)

[2 (a + b + c)] .
Hay
ab
c(b + c)
+

bc
a(c + a)
+
ac
b(a + b)

3
2
.
Ta suy ra
1 + a
1 − a
+
1 + b
1 − b
+
1 + c
1 − c
≤ 2

b
a
+
c
b
+
a
c

.

Bất đẳng thức (1.1) được chứng minh.
7
Ví dụ 1.2 (IMO 1995 - [5]). Cho các số dương a, b, c thỏa mãn điều kiện
abc = 1. Chứng minh rằng
1
a
3
(b + c)
+
1
b
3
(c + a)
+
1
c
3
(a + b)

3
2
. (1.2)
Nhận xét 1.2. Vai trò của a, b, c trong bất đẳng thức là như nhau. Mặt
khác vế trái của bất đẳng thức chứa a
2
, b
2
, c
2
và a, b, c nhưng không có bậc

hai dạng ab, bc, ca. Do đó ta đặt ẩn phụ cho ab, bc, ca.
Giải
Đặt
bc = x, ca = y, ab = z.
Khi đó theo giả thiết abc = 1 ta được
1
a
= x,
1
b
= y,
1
c
= z.
Suy ra
a =
1
x
, b =
1
y
, c =
1
z
.
Áp dụng bất đẳng thức AM-GM, kết hợp với giả thiết abc = 1 ta có
x + y + z =
1
a
+

1
b
+
1
c
≥ 3
3

1
abc
= 3.
Biến đổi bất đẳng thức (1.2) ta có
(1.2) ⇔
x
3
1
y
+
1
z
+
y
3
1
z
+
1
x
+
z

3
1
x
+
1
y

3
2

x
3
.yz
y + z
+
y
3
.zx
z + x
+
z
3
.xy
x + y

3
2

x
2

y + z
+
y
2
z + x
+
z
2
x + y

3
2
.
Do vai trò của a, b, c cũng như của x, y, z là như nhau, không làm mất
tính tổng quát, ta có thể giả thiết
a ≥ b ≥ c > 0.
Suy ra
0 < x ≤ y ≤ z.
8
Khi đó







x
2
≤ y

2
≤ z
2
1
y + z

1
z + x

1
x + y
.
Suy ra
x
2
y + z
+
y
2
z + x
+
z
2
x + y

x
2
y + x
+
y

2
z + y
+
z
2
x + z

x
2
y + z
+
y
2
z + x
+
z
2
x + y

x
2
x + z
+
y
2
y + x
+
z
2
z + y

.
Cộng vế với vế ta được
x
2
y + z
+
y
2
z + x
+
z
2
x + y

1
2

x
2
+ z
2
x + z
+
y
2
+ x
2
y + x
+
z

2
+ y
2
z + y

.
Theo bất đẳng thức AM-GM thì
x
2
+ z
2

(x + z)
2
2
y
2
+ x
2

(y + x)
2
2
z
2
+ y
2

(z + y)
2

2
và kết hợp với x + y + z ≥ 3 ta được
1
2

x
2
+ z
2
x + z
+
y
2
+ x
2
y + x
+
z
2
+ y
2
z + y


1
2
(x + y + z) ≥
3
2
.

Suy ra
x
2
y + z
+
y
2
z + x
+
z
2
x + y

3
2
.
Vậy
1
a
3
(b + c)
+
1
b
3
(c + a)
+
1
c
3

(a + b)

3
2
.
Bất đẳng thức (1.2) được chứng minh.
9
1.2.2 Phương pháp phản chứng
• Ý tưởng
Phương pháp phản chứng để chứng minh bất đẳng thức thường dùng
khi điều kiện bài toán thì phức tạp, còn bất đẳng thức cần chứng minh
thì đơn giản. Khi đó ta đảo điều kiện và kết luận của bài toán cho nhau.
Muốn chứng minh bất đẳng thức A ≥ B bằng phương pháp phản
chứng, ta giả sử A < B. Bằng lập luận ta suy ra điều mâu thuẫn. Do đó
điều giả sử là sai. Vậy nên bất đẳng thức A ≥ B đúng.
• Ví dụ minh họa
Ví dụ 1.3 (IMO 2001 - [1]). Cho a, b, c là các số thực dương. Chứng minh
rằng
a

a
2
+ 8bc
+
b

b
2
+ 8ca
+

c

c
2
+ 8ab
≥ 1. (1.3)
Nhận xét 1.3. Để mất dấu căn thức ở vế trái của bất đẳng thức ta đặt
ẩn phụ cho từng số hạng.
Giải
Đặt
x =
a

a
2
+ 8bc
y =
b

b
2
+ 8ca
z =
c

c
2
+ 8ab
.
Vì a, b, c là các số thực dương nên x, y, z cũng là các số thực dương.

Mặt khác ta có
x =
a

a
2
+ 8bc
<
a

a
2
= 1
y =
b

b
2
+ 8ca
<
b

b
2
= 1
z =
c

c
2

+ 8ab
<
c

c
2
= 1.
Do đó x, y, z ∈ (0; 1).
10
Từ cách đặt x, y, z ta được
a
2
8bc
=
x
2
1 − x
2
b
2
8ca
=
y
2
1 − y
2
c
2
8ab
=

z
2
1 − z
2
.
Nhân vế với vế các đẳng thức trên ta được
1
512
=
x
2
1 − x
2
·
y
2
1 − y
2
·
z
2
1 − z
2
Suy ra
(1 − x
2
)(1 − y
2
)(1 − z
2

) = 512x
2
y
2
z
2
.
Như vậy ta cần chứng minh
x + y + z ≥ 1
trong đó
x, y, z ∈ (0; 1)

(1 − x
2
)(1 − y
2
)(1 − z
2
) = 512x
2
y
2
z
2
.
Ta dùng phương pháp phản chứng để chứng minh bất đẳng thức trên.
Giả sử
x + y + z < 1.
Vì x + y + z > 0 nên bình phương 2 vế bất đẳng trên ta được
(x + y + z)

2
< 1.
Do đó
(1 − x
2
)(1 − y
2
)(1 − z
2
)
>

(x + y + z)
2
− x
2

(x + y + z)
2
− y
2

(x + y + z)
2
− z
2

= (y + z)(y + z + 2x).(z + x)(z + x + 2y).(x + y)(x + y + 2z).
Áp dụng bất đẳng thức AM - GM ta có
(y + z)(y + z + 2x).(z + x)(z + x + 2y).(x + y)(x + y + 2z)

≥ 2

yz.4
4

yzx
2
.2

zx.4
4

zxy
2
.2

xy.4
4

xyz
2
= 512x
2
y
2
z
2
.
11
Như vậy

(1 − x
2
)(1 − y
2
)(1 − z
2
) > 512x
2
y
2
z
2
.
Điều này mâu thuẫn với
(1 − x
2
)(1 − y
2
)(1 − z
2
) = 512x
2
y
2
z
2
.
Vậy điều giả sử là sai.
Suy ra
x + y + z ≥ 1.

Hay
a

a
2
+ 8bc
+
b

b
2
+ 8ca
+
c

c
2
+ 8ab
≥ 1.
Bất đẳng thức được chứng minh.
1.2.3 Phương pháp quy nạp toán học
• Ý tưởng
Muốn chứng minh một bất đẳng thức đúng với mọi n ≥ n
0
bằng phương
pháp quy nạp toán học, ta thực hiện các bước sau
- Kiểm tra bất đẳng thức đúng với n = n
0
.
- Giả thiết bất đẳng thức đúng với n = k với k > n

0
(gọi là giả thiết
quy nạp), rồi chứng minh bất đẳng thức đúng với n = k + 1.
- Kết luận.
• Ví dụ minh họa
Ví dụ 1.4 (Bất đẳng thức Bernoulli). Chứng minh rằng với mọi bộ số
thực a
1
, a
2
, . . . , a
n
cùng dấu và lớn hơn −1, ta đều có
n

i=1
(1 + a
i
) ≥ 1 +
n

i=1
a
i
. (1.4)
Giải
- Với n = 1 thay vào (1.4) ta được 1 + a
1
≥ 1 + a
1

.
Suy ra bất đẳng thức (1.4) đúng với n = 1.
12
- Giả sử bất đẳng thức (1.4) đúng với n = k, tức là
k

i=1
(1 + a
i
) ≥ 1 +
k

i=1
a
i
.
Ta phải chứng minh bất đẳng thức (1.4) đúng với n = k + 1, tức là
k+1

i=1
(1 + a
i
) ≥ 1 +
k+1

i=1
a
i
.
Ta có

k+1

i=1
(1 + a
i
) = (1 + a
k+1
)
k

i=1
(1 + a
i
)
≥ (1 + a
k+1
)

1 +
k

i=1
a
i

(theo giả thiết quy nạp)
=

1 +
k


i=1
a
i
+ a
k+1

+ a
k+1
k

i=1
a
i
=

1 +
k+1

i=1
a
i

+ a
k+1
k

i=1
a
i

.
Vì a
1
, a
2
, . . . , a
k
, a
k+1
cùng dấu và lớn hơn −1 nên
a
k+1
k

i=1
a
i
≥ 0.
Do đó
k+1

i=1
(1 + a
i
) ≥ 1 +
k+1

i=1
a
i

.
Suy ra (1.4) đúng với n = k + 1. Vậy bất đẳng thức được chứng minh.
Dấu đẳng thức xảy ra khi và chỉ khi a
1
= a
2
= ··· = a
n
.
1.2.4 Sử dụng tam thức bậc hai
Bất đẳng thức cơ bản và cũng là quan trọng nhất trong chương trình
đại số bậc trung học phổ thông là bất đẳng thức
x
2
≥ 0, ∀x ∈ R.
13
Dấu đẳng thức xảy ra khi và chỉ khi x = 0.
Ta cũng thường gặp bất đẳng thức dạng sau
(x
1
− x
2
)
2
≥ 0, ∀x
1
, x
2
∈ R
hay

x
2
1
+ x
2
2
≥ 2x
1
x
2
, ∀x
1
, x
2
∈ R.
Dấu đẳng thức xảy ra khi và chỉ khi x
1
= x
2
.
Định lý 1.1. Xét tam thức bậc hai f (x) = ax
2
+ bx + c, a = 0, ∆ =
b
2
− 4ac.
i) Nếu ∆ < 0 thì af(x) > 0, ∀x ∈ R.
ii) Nếu ∆ = 0 thì af (x) ≥ 0, ∀x ∈ R.
Dấu đẳng thức xảy ra khi và chỉ khi x = −
b

2a
.
iii) Nếu ∆ > 0 thì af(x) = a
2
(x − x
1
)(x − x
2
), với
x
1,2
= −
b
2a



2 | a |
.
Trong trường hợp này, af(x) < 0 khi x ∈ (x
1
; x
2
) và af(x) > 0 khi
x < x
1
hoặc x > x
2
.
Định lý 1.2 (Định lí đảo). Điều kiện cần và đủ để tồn tại số α sao cho

af(α) < 0 là ∆ > 0 và x
1
< α < x
2
, trong đó x
1
, x
2
là các nghiệm của
f(x) xác định theo công thức
x
1,2
= −
b
2a



2 | a |
.
• Ý tưởng vận dụng
Để chứng minh bất đẳng thức A > B bằng phương pháp sử dụng tam
thức bậc hai ta làm như sau
- Viết biểu thức A − B thành tam thức bậc hai theo một biến số nào
đó.
- Tính ∆.
- Biện luận theo định lí để suy ra bất đẳng thức cần chứng minh.
14
• Ví dụ minh họa
Ví dụ 1.5. Cho các số thực dương a, b, c. Chứng minh rằng

a
2
+ b
2
+ c
2
+ 2abc + 1 ≥ 2(ab + bc + ca). (1.5)
Giải
Chuyển vế bất đẳng thức (1.5) ta có
(1.5) ⇔ a
2
+ b
2
+ c
2
+ 2abc + 1 − 2(ab + bc + ca) ≥ 0
⇔ a
2
+ 2a(bc − b − c) + (b − c)
2
+ 1 ≥ 0.
Đặt
f(a) = a
2
+ 2a(bc − b − c) + (b − c)
2
+ 1.
* Nếu bc −b −c ≥ 0, hay (b − 1)(c − 1) ≥ 1.
Kết hợp với giả thiết a dương ta được
f(a) = a

2
+ 2a(bc − b − c) + (b − c)
2
+ 1 ≥ 0.
* Nếu bc − b − c < 0, hay (b − 1)(c − 1) < 1 ta tính biệt thức ∆

của
f(a)


= (bc − b − c)
2
− (b − c)
2
− 1
= b
2
c
2
− 2b
2
c − 2bc
2
+ 4bc − 1
= bc(b − 2)(c − 2) − 1.
Vì ta xét (b − 1)(c − 1) < 1 với a, b, c là các số thực dương nên chỉ
cần biện luận ∆

của f(a) theo hai trường hợp sau
- Trường hợp 1: có đúng một trong hai số b, c lớn hơn 2, số còn lại

không lớn hơn 2. Khi đó ta có
(b − 2)(c − 2) ≤ 0.
Từ đó suy ra


= bc(b − 2)(c − 2) − 1 < 0.
- Trường hợp 2: cả hai số b, c đều không lớn hơn 2. Khi đó áp dụng
bất đẳng thức AM-GM ta có


= bc(b −2)(c −2)−1 ≤

b + c + (2 − b) + (2 − c)
4

4
−1 = 0.
15
Tóm lại trong mọi trường hợp ta đều chứng minh được ∆

≤ 0.
Áp dụng định lí về dấu của tam thức bậc hai ta có
1.f(a) ≥ 0.
Suy ra
f(a) = a
2
+ 2a(bc − b − c) + (b − c)
2
+ 1 ≥ 0.
Hay

a
2
+ b
2
+ c
2
+ 2abc + 1 ≥ 2(ab + bc + ca).
Bất đẳng thức (1.5) được chứng minh.
Dấu đẳng thức xảy ra khi và chỉ khi a = b = c = 1.
Ví dụ 1.6 (Bất đẳng thức Bunhiacovski). Chứng minh rằng với mọi bộ
số (a
i
), (b
i
) ta luôn có bất đẳng thức sau

n

i=1
a
i
b
i

2


n

i=1

a
2
i

n

i=1
b
2
i

. (1.6)
Giải
Ta viết bất đẳng thức (1.6) dưới dạng
(a
1
b
1
+ a
2
b
2
+ ··· + a
n
b
n
)
2
≤ (a
2

1
+ a
2
2
+ ··· + a
2
n
)(b
2
1
+ b
2
2
+ ··· + b
2
n
).
Xét tam thức
f(x) = (a
1
x − b
1
)
2
+ (a
2
x − b
2
)
2

+ ··· + (a
n
x − b
n
)
2
.
Dễ thấy f (x) ≥ 0, ∀x ∈ R.
Khai triển f (x) ta có
f(x) = (a
2
1
+ a
2
2
+ ··· + a
2
n
)x
2
− 2(a
1
b
1
+ a
2
b
2
+ ··· + a
n

b
n
)x
+ (b
2
1
+ b
2
2
+ ··· + b
2
n
).
* Nếu a
2
1
+ a
2
2
+ ··· + a
2
n
= 0.
Hay a
1
= a
2
= ··· = a
n
= 0, khi đó bất đẳng thức (1.6) có dạng

0 ≤ 0 (luôn đúng).
* Nếu a
2
1
+ a
2
2
+ ··· + a
2
n
= 0.
16


f
= (a
1
b
1
+ a
2
b
2
+ ···+ a
n
b
n
)
2
−(a

2
1
+ a
2
2
+ ···+ a
2
n
)(b
2
1
+ b
2
2
+ ···+ b
2
n
).

f(x) ≥ 0, ∀x ∈ R

a
2
1
+ a
2
2
+ ··· + a
2
n

> 0, với ∀a
i
, i = 1, . . . , n
nên


f
< 0, ∀x ∈ R.
Suy ra
(a
1
b
1
+ a
2
b
2
+ ··· + a
n
b
n
)
2
− (a
2
1
+ a
2
2
+ ··· + a

2
n
)(b
2
1
+ b
2
2
+ ··· + b
2
n
) < 0
hay
(a
1
b
1
+ a
2
b
2
+ ··· + a
n
b
n
)
2
< (a
2
1

+ a
2
2
+ ··· + a
2
n
)(b
2
1
+ b
2
2
+ ··· + b
2
n
).
Suy ra

n

i=1
a
i
b
i

2


n


i=1
a
2
i

n

i=1
b
2
i

.
Dấu đẳng thức xảy ra khi và chỉ khi
a
1
b
1
=
a
2
b
2
= ··· =
a
n
b
n
hay a

i
= kb
i
, i = 1, . . . , n.
1.2.5 Sử dụng bất đẳng thức AM-GM
Bất đẳng thức AM-GM hay gọi ngắn gọn hơn là bất đẳng thức AG là
bất đẳng thức giữa giá trị trung bình cộng và trung bình nhân. Bất đẳng
thức AM-GM được ba nhà toán học Schwarz, Bunhiacovski và Cauchy
phát minh ra. Tuy nhiên bất đẳng thức này vẫn được gọi là bất đẳng thức
Cauchy nhiều hơn và thường nhầm lẫn rằng Cauchy tìm ra bất đẳng thức
này. Tuy nhiên Cauchy là người đưa ra cách chứng minh bất đẳng thức
AM-GM rất hay và độc đáo. Tên gọi bất đẳng thức AM-GM là chuẩn quốc
tế được viết tắt từ Arithmetic Mean-Geometric Mean.
Định lý 1.3. Giả sử x
1
, x
2
, . . . x
n
là các số không âm. Khi đó
x
1
+ x
2
+ ··· + x
n
n

n


x
1
.x
2
. . . x
n
.
17
Dấu đẳng thức xảy ra khi và chỉ x
1
= x
2
= ··· = x
n
.
Để áp dụng tốt bất đẳng thức AM-GM ta phải nghiên cứu kĩ điều kiện
xảy ra dấu đẳng thức và áp dụng kĩ thuật chọn “điểm rơi”.
Chứng minh. Từ hệ thức bậc hai
u
2
1
+ u
2
2
≥ 2u
1
u
2
, ∀u
1

, u
2
∈ R,
ta suy ra
x
1
+ x
2
2


x
1
x
2
, ∀x
1
, x
2
không âm.
Thay biến mới
x
1
+ x
2
2

x
3
+ x

4
2
vào công thức trên ta được
x
1
+ x
2
+ x
3
+ x
4
4


x
1
+ x
2
2
·
x
3
+ x
4
2

1
2

4


x
1
x
2
x
3
x
4
.
Tiếp tục quá trình như trên ta thấy định lí đúng với n = 2; 4; . . . và nói
chung, đúng với n là lũy thừa của 2. Đây chính là quy nạp theo hướng lên
trên.
Bây giờ ta thực hiện quy trình quy nạp theo hướng xuống phía dưới.
Ta chứng minh rằng khi định lí đúng với n (n > 1) thì nó cũng đúng với
n − 1. Ta thay x
n
trong định lí bởi
x
1
+ x
2
+ ··· + x
n−1
n − 1
và giữ nguyên các biến x
i
khác ta thu được
x
1

+ x
2
+ ··· + x
n−1
+
x
1
+ x
2
+ ··· + x
n−1
n − 1
n


n

x
1
x
2
. . . x
n−1
.
n

x
1
+ x
2

+ ··· + x
n−1
n − 1
.
Từ đây ta suy ra
x
1
+ x
2
+ ··· + x
n−1
n − 1

n−1

x
1
.x
2
. . . x
n−1
.
Từ kết quả đã chứng minh theo cặp hướng (lên - xuống), ta thu được
phép chứng minh của định lí.
18
• Ví dụ minh họa
Ví dụ 1.7 (VMO 1996). Cho các số thực không âm a, b, c thỏa mãn
ab + bc + ca + abc = 4.
Chứng minh rằng
a + b + c ≥ ab + bc + ca. (1.7)

Giải
Từ giả thiết a, b, c là các số thực không âm thỏa mãn
ab + bc + ca + abc = 4
ta có thể đặt
a =
2x
y + z
, b =
2y
z + x
, c =
2z
x + y
với x, y, z là các số thực dương.
Khi đó, bất đẳng thức (1.7) được viết lại thành
x
y + z
+
y
z + x
+
z
x + y

2xy
(x + z)(y + z)
+
2yz
(y + x)(z + x)
+

2yz
(z + y)(x + y)
.
Áp dụng bất đẳng thức AM-GM ta có
2xy
(x + z)(y + z)
= xy ·
2
(x + z)(y + z)
≤ xy ·

1
(x + z)
2
+
1
(y + z)
2

2yz
(y + x)(z + x)
= yz ·
2
(y + x)(z + x)
≤ yz ·

1
(y + x)
2
+

1
(z + x)
2

2zx
(z + y)(x + y)
= zx ·
2
(z + y)(x + y)
≤ zx ·

1
(z + y)
2
+
1
(x + y)
2

.
Do đó ta có
V P =
2xy
(x + z)(y + z)
+
2yz
(y + x)(z + x)
+
2zx
(z + y)(x + y)

≤ xy

1
(x + z)
2
+
1
(y + z)
2

+ yz

1
(y + x)
2
+
1
(z + x)
2

+
+ zx

1
z + y)
2
+
1
(x + y)
2


=
xy
(x + z)
2
+
xy
(y + z)
2
+
yz
(y + x)
2
+
yz
(z + x)
2
+
zx
(z + y)
2
+
zx
(x + y)
2
=
xy + yz
(x + z)
2
+

xy + zx
(y + z)
2
+
yz + zx
(y + x)
2
=
x
y + z
+
y
z + x
+
z
x + y
.
19
Bất đẳng thức được chứng minh.
Đẳng thức xảy ra khi và chỉ khi x = y = z hay a = b = c = 1.
1.2.6 Sử dụng bất đẳng thức Bunhiacovski
Tại Việt Nam và một số nước Đông Âu, bất đẳng thức Bunhiacovski,
hay còn gọi là bất đẳng thức Cauchy-Schwarz hoặc Cauchy-Bunhiacovski
được phát biểu như sau
Định lý 1.4. Với mọi bộ số (a
i
), (b
i
) ta luôn có bất đẳng thức sau


n

i=1
a
i
b
i

2


n

i=1
a
2
i

n

i=1
b
2
i

.
Dấu đẳng thức xảy ra khi và chỉ khi hai bộ số (a
i
) và (b
i

) tỉ lệ với nhau,
tức là tồn tại cặp số thực α, β không đồng thời bằng 0, sao cho
αa
i
+ βb
i
= 0, ∀i = 1, 2, . . . , n.
Theo cách gọi của các chuyên gia đầu ngành về bất đẳng thức (Hardy
G.H., Littlewood J.E., Polya G., Bellman R., ) thì bất đẳng thức tích
phân mới mang tên là bất đẳng thức Bunhiacovski.
Định lý 1.5. Với mọi cặp hàm f(x) và g(x) liên tục trên [a; b] ta đều có


b
a
f(x).g(x)dx

2


b
a
f
2
(x)dx.

b
a
g
2

(x)dx.
• Ví dụ minh họa
Ví dụ 1.8 ([8]). Cho a, b, c và x, y, z là các số không âm thỏa mãn
a + b + c = x + y + z.
Chứng minh rằng
ax(a + x) + by(b + y) + cz(c + z) ≥ 3(abc + xyz). (1.8)
Giải
Áp dụng bất đẳng thức Bunhiacovski cho hai cặp số

a

x, b

y, c

z




yz,

zx,

xy

20
ta được

a

2
x + b
2
y + c
2
z

(yz + zx + xy)
≥ (a

xyz + b

xyz + c

xyz)
2
= xyz (a + b + c)
2
.
Từ giả thiết
a + b + c = x + y + z
kết hợp với bất đẳng thức AM-GM ta có
(a + b + c)
2
= (x + y + z)
2
≥ 3(xy + yz + zx).
Từ đó ta suy ra

a

2
x + b
2
y + c
2
z

(yz + zx + xy) ≥ 3xyz(xy + yz + zx).
Hay
a
2
x + b
2
y + c
2
z ≥ 3xyz.
Chứng minh tương tự ta được
ax
2
+ by
2
+ cz
2
≥ 3abc.
Cộng vế với vế của hai bất đẳng thức trên ta được
a
2
x + b
2
y + c

2
z + ax
2
+ by
2
+ cz
2
≥ 3xyz + 3abc.
Hay
ax(a + x) + by(b + y) + cz(c + z) ≥ 3(abc + xyz).
Bất đẳng thức được chứng minh.
Ví dụ 1.9 ([5]). Cho hàm số liên tục
f : [0; 1] −→ [−1; 1].
Chứng minh rằng

1
0

1 − f
2
(x)dx ≤

1 −


1
0
f(x)dx

2

. (1.9)
21
Nhận xét 1.4. Để áp dụng bất đẳng thức Bunhiacovski dạng tích phân
ta thực hiện hai bước sau
- Bước 1. Xác định cặp hàm số f(x) và g(x).
- Bước 2. Vận dụng định lí để chứng minh bất đẳng thức.
Giải
Áp dụng bất đẳng thức Bunhiacovski cho cặp hàm số
H(x) =

1 − f
2
(x), G(x) ≡ 1
trên [0; 1] ta có


1
0

1 − f
2
(x)dx

2


1
0

1 − f

2
(x)

dx.

1
0
dx.
Suy ra

1
0

1 − f
2
(x)dx ≤

1 −

1
0
f
2
(x)dx.
Tiếp theo, áp dụng bất đẳng thức Bunhiacovski cho cặp hàm số
F (x) ≡ f (x), G(x) ≡ 1
trên [0; 1] ta có


1

0
f(x)dx

2


1
0
f
2
(x)dx.

1
0
dx.
Hay


1
0
f(x)dx

2


1
0
f
2
(x)dx.

Từ đó suy ra
1 −


1
0
f(x)dx

2
≥ 1 −

1
0
f
2
(x)dx.
Vậy nên

1
0

1 − f
2
(x)dx ≤

1 −


1
0

f(x)dx

2
.
Ta được điều cần chứng minh.
22
1.2.7 Sử dụng bất đẳng thức Karamata
Định nghĩa 1.2. Hàm số y = f(x) được gọi là hàm lồi (xuống phía dưới)
trong khoảng (a; b) nếu với mọi a < x
1
, x
2
< b và mọi α ∈ (0; 1) luôn có
bất đẳng thức
αf(x
1
) + (1 − α)f (x
2
) ≥ f (αx
1
+ (1 − α)x
2
) .
Định nghĩa 1.3. Hàm số y = f(x) được gọi là hàm lõm (lên phía trên)
trong khoảng (a; b) nếu với mọi a < x
1
, x
2
< b và mọi α ∈ (0; 1) luôn có
bất đẳng thức

αf(x
1
) + (1 − α)f (x
2
) ≤ f (αx
1
+ (1 − α)x
2
) .
Định lý 1.6 (Bất đẳng thức Karamata). Cho hai dãy số x
k
, y
k
∈ I(a, b),
k = 1, 2, . . . , n, thỏa mãn các điều kiện
x
1
≤ x
2
≤ ··· ≤ x
n
, y
1
≤ y
2
≤ ··· ≤ y
n









x
1
≥ y
1
x
1
+ x
2
≥ y
1
+ y
2
. . . . . . . . . . . . . . . . . . . . .
x
1
+ x
2
+ ··· + x
n−1
≥ y
1
+ y
2
+ ··· + y
n−1

x
1
+ x
2
+ ··· + x
n
≥ y
1
+ y
2
+ ··· + y
n
Khi đó, ứng với mỗi hàm lồi thực sự f(x)(f

(x) > 0) trên I(a, b), ta đều

f(x
1
) + f(x
2
) + ··· + f(x
n
) ≥ f(y
1
) + f(y
2
) + ··· + f(y
n
).
Chứng minh. Sử dụng biểu diễn đối với hàm lồi

f(x
1
) + f(x
2
) + ···+ f(x
n
) = max
t
1
, ,t
n
∈I(a,b)

n

i=1
f(t
i
) +
n

i=1
(x
i
− t
i
)f

(t
i

)

Không mất tổng quát, ta giả sử bộ số t
1
, . . . , t
n
∈ I(a, b) cũng là một bộ
số giảm, tức là
t
1
≥ t
2
≥ ··· ≥ t
n
.
Khi đó, để chứng minh
f(x
1
) + f(x
2
) + ···+ f(x
n
) = max
t
1
, ,t
n
∈I(a,b)

n


i=1
f(t
i
) +
n

i=1
(x
i
− t
i
)f

(t
i
)

23
ta chỉ cần chứng minh rằng
x
1
f

(t
1
) + x
2
f


(t
2
) + ··· + x
n
f

(t
n
) ≥ y
1
f

(t
1
) + y
2
f

(t
2
) + ··· + y
n
f

(t
n
).
Sử dụng biến đổi Abel
x
1

f

(t
1
) + x
2
f

(t
2
) + ··· + x
n
f

(t
n
) = S
1
[f

(t
1
) − f

(t
2
)]+
+ S
2
[f


(t
2
) − f

(t
3
)] + ··· + S
n−1
[f

(t
n−1
) − f

(t
n
)] + S
n
f

(t
n
).
với
S
k
(x) := x
1
+ x

2
+ ··· + x
k
.
Vì rằng f

(x) > 0 nên f

(x
k
) ≤ f

(x
k−1
). Mặt khác, do
S
k
(x) ≥ S
k
(y)(k = 1, 2, . . . , n − 1) và S
n
(x) = S
n
(y), ta thu được ngay
x
1
f

(t
1

) + x
2
f

(t
2
) + ··· + x
n
f

(t
n
) ≥ y
1
f

(t
1
) + y
2
f

(t
2
) + ··· + y
n
f

(t
n

).
• Ví dụ minh họa
Ví dụ 1.10 ([6]). Cho các số dương a, b, c thỏa mãn a + b + c = 1. Chứng
minh rằng
T =
3

2
a
+ 9bc +
3

2
b
+ 9ca +
3

2
c
+ 9ab ≤
3

6
abc
+ 27. (1.10)
Giải
Vì f(x) =
3

x, x > 0 là hàm lõm nên ta có

3

2
a
+ 9bc +
3

2
b
+ 9ca +
3

2
c
+ 9ab ≤
3
3

3
3

2
a
+
2
b
+
2
c
+ 9(ab + bc + ca)

hay
3

2
a
+ 9bc +
3

2
b
+ 9ca +
3

2
c
+ 9ab ≤
3
3

3
3


2
abc
+ 9

(ab + bc + ca).
Vì 1 = (a + b + c)
2

≥ 3(ab + bc + ca)
nên T ≤
3

6
abc
+ 27.
Ví dụ 1.11 (IMO 2000). Cho a, b, c là các số thực dương thỏa mãn điều
kiện abc = 1. Chứng minh rằng

a − 1 +
1
b

+

b − 1 +
1
c

+

c − 1 +
1
a

≤ 1. (1.11)
24
Giải
Vì abc = 1 nên tồn tại các số dương x, y, z sao cho a =

x
y
, b =
y
z
, c =
z
x
.
Bất đẳng thức cần chứng minh trở thành
(x − y + z)(y − z + x)(z − x + y) ≤ xyz.
Ta để ý rằng, (x − y + z) + (y − z + x) = 2x > 0, do đó, trong ba số
x −y +z, y −z +x, z −x +y không thể có trường hợp hai số cùng âm. Nếu
trong ba số trên có một hoặc ba số âm, hiển nhiên ta có bất đẳng thức cần
chứng minh. Trường hợp cả ba số đều dương, bằng cách lấy logarit hai vế,
ta có
ln(x − y + z) + ln(y − z + x) + ln(z − x + y) ≤ ln x + ln y + ln z.
Vì f(x) = ln x là hàm lõm trên (0, +∞), do đó, sử dụng bất đẳng thức
Karamata, ta được
ln(y − z + x) + ln(x − y + z) + ln(z − x + y) ≤ ln x + ln y + ln z.
Đẳng thức xảy ra khi và chỉ khi x = y = z hay a = b = c = 1.
1.2.8 Vận dụng tính chất của hàm số đơn điệu vào chứng minh
bất đẳng thức
Định lý 1.7. Cho hàm số f (x) có đạo hàm trên khoảng (a, b).
(i) Nếu f

(x) > 0 với mọi x ∈ (a; b) thì hàm số f(x) đồng biến trên
khoảng đó.
(ii) Nếu f


(x) < 0 với mọi x ∈ (a; b) thì hàm số f (x) nghịch biến trên
khoảng đó.
• Ý tưởng vận dụng
- Biến đổi bất đẳng thức để làm xuất hiện hàm số f(x) nào đó trên
khoảng xác định.
- Tính f

(x).
- Biện luận theo định lí để suy ra bất đẳng thức cần chứng minh.

×